Calculus: trig functions anti-derivatives

Click For Summary
The discussion revolves around finding the function g(0) given the derivative g'(x) and the condition g(1)=143/9. The user successfully derived the antiderivative but failed to account for the constant of integration, which is essential for determining the correct function. It was suggested that the user should include this constant, C, in their expression for g(x) to satisfy the condition provided. Additionally, there may be issues related to calculator settings, such as using degrees versus radians, which could affect the calculations. Properly incorporating these elements will help resolve the discrepancies in the solution.
Melawrghk
Messages
140
Reaction score
0

Homework Statement


Let g'(x)=-17x16* sin(Ax9) - 9Ax25*cos(Ax9)

g(1)=143/9
Where A is a real number such that tanA=1/sqrt(80), 0<A<pi/2

Find g(0)

The Attempt at a Solution


I was able to get the antiderivative of g'(x), so that:
g(x) = -x17*sin(Ax9)

I got A=6.3794 degrees.

BUT, I don't get g(1)=143/9

What am I doing wrong? Did I assume wrong that the g'(x) was created by the product rule?
 
Physics news on Phys.org
Melawrghk said:

Homework Statement


Let g'(x)=-17x16* sin(Ax9) - 9Ax25*cos(Ax9)

g(1)=143/9
Where A is a real number such that tanA=1/sqrt(80), 0<A<pi/2

Find g(0)

The Attempt at a Solution


I was able to get the antiderivative of g'(x), so that:
g(x) = -x17*sin(Ax9)

I got A=6.3794 degrees.

BUT, I don't get g(1)=143/9

What am I doing wrong? Did I assume wrong that the g'(x) was created by the product rule?

Your antiderivative looks fine. Did you remember to add a constant? IOW, you should have
g(x) = -x17*sin(Ax9) + C

You're sort of given A, and you're given that g(1) = 143/9, so you can find C. If you did all that, you can still run into problems calculating things with a calculator, such as when it's in radian mode but you're working with degrees.
 
Question: A clock's minute hand has length 4 and its hour hand has length 3. What is the distance between the tips at the moment when it is increasing most rapidly?(Putnam Exam Question) Answer: Making assumption that both the hands moves at constant angular velocities, the answer is ## \sqrt{7} .## But don't you think this assumption is somewhat doubtful and wrong?

Similar threads

  • · Replies 1 ·
Replies
1
Views
1K
  • · Replies 2 ·
Replies
2
Views
1K
  • · Replies 3 ·
Replies
3
Views
2K
  • · Replies 11 ·
Replies
11
Views
3K
Replies
2
Views
2K
  • · Replies 10 ·
Replies
10
Views
2K
  • · Replies 5 ·
Replies
5
Views
2K
  • · Replies 1 ·
Replies
1
Views
2K
Replies
8
Views
2K
  • · Replies 4 ·
Replies
4
Views
2K